Difference between revisions of "2011 AMC 12A Problems/Problem 12"

(Created page with '== Problem == == Solution == == See also == {{AMC12 box|year=2011|num-b=11|num-a=13|ab=A}}')
 
(Problem)
Line 1: Line 1:
 
== Problem ==
 
== Problem ==
 +
A power boat and a raft both left dock <math>A</math> on a river and headed downstream. The raft drifted at the speed of the river current. The power boat maintained a constant speed with respect to the river. The power boat reached dock <math>B</math> downriver, then immediately turned and traveled back upriver. It eventually met the raft on the river 9 hours after leaving dock <math>A.</math> How many hours did it take the power raft to go from <math>A</math> to <math>B?</math>
 +
 +
<math>
 +
\textbf{(A)}\ 3 \qquad
 +
\textbf{(B)}\ 3.5 \qquad
 +
\textbf{(C)}\  4 \qquad
 +
\textbf{(D)}\ 4.5 \qquad
 +
\textbf{(E)}\ 5 </math>
 +
 
== Solution ==
 
== Solution ==
 
== See also ==
 
== See also ==
 
{{AMC12 box|year=2011|num-b=11|num-a=13|ab=A}}
 
{{AMC12 box|year=2011|num-b=11|num-a=13|ab=A}}

Revision as of 02:34, 10 February 2011

Problem

A power boat and a raft both left dock $A$ on a river and headed downstream. The raft drifted at the speed of the river current. The power boat maintained a constant speed with respect to the river. The power boat reached dock $B$ downriver, then immediately turned and traveled back upriver. It eventually met the raft on the river 9 hours after leaving dock $A.$ How many hours did it take the power raft to go from $A$ to $B?$

$\textbf{(A)}\ 3 \qquad \textbf{(B)}\ 3.5 \qquad \textbf{(C)}\  4 \qquad \textbf{(D)}\ 4.5 \qquad \textbf{(E)}\ 5$

Solution

See also

2011 AMC 12A (ProblemsAnswer KeyResources)
Preceded by
Problem 11
Followed by
Problem 13
1 2 3 4 5 6 7 8 9 10 11 12 13 14 15 16 17 18 19 20 21 22 23 24 25
All AMC 12 Problems and Solutions